You are on page 1of 10

TỔNG HỢP MỘT SỐ BÀI ĐA THỨC, DÃY SỐ TRONG

KỲ THI CÁC NƯỚC NĂM 2017


Bài 1. (USA Preselection)
Cho các đa thức P, Q hệ số thực thỏa mãn

 P( x)  Q( x) với mọi x.

Chứng minh rằng P( x)  Q( x).

Lời giải.
Đặt R( x)  P( x)  Q( x) thì dễ thấy 1  R( x)  1 với mọi x.

Nếu deg R  1 thì điều trên không thể xảy ra nên deg R  0.

Đặt R( x)  c và giả sử c  0. Không mất tính tổng quát, ta có thể xét c  0 , thì

P ( x )  Q( x )  c .

Ta thấy, tồn tại x0 đủ lớn thì P, Q đều đơn điệu và song ánh trên ( x0 ;  ) .

Xét m  x0 sao cho P ( m)  k    , khi đó Q(m)  P(m)  c  k nên

Q( x)   P(m) , mâu thuẫn.


Vậy c  0 hay P( x)  Q( x).

Bài 2. (Bosnita)

 
Cho    0;  thỏa mãn tan   2. Xét dãy số (un ) xác định bởi
 2

un  tan n với n  1, 2,3,

Chứng minh rằng với mọi n thì un có thể viết thành phân số nguyên tối giản có mẫu là số lẻ.

Gợi ý. Sử dụng công thức


tan(n  1)  tan  tan( n  1)  2
tan n  tan((n  1)   )   .
1  tan(n  1) tan  1  2 tan(n  1)

p
Quy nạp, đặt tan(n  1)  với ( p, q)  1 và q lẻ; thay vào là có đpcm.
q

Bài 3. (Indonesia)

1
Cho đa thức P( x) có hệ số nguyên và có ít nhất 9 nghiệm nguyên phân biệt. Giả sử rằng tồn tại
x0   sao cho P ( x0 )  2017 . Chứng minh rằng P ( x0 )  0.

Lời giải.
Đặt P ( x )  ( x  x1 )( x  x2 )  ( x  x9 )Q ( x) với Q( x)  [ x] , trong đó x1 , x2 , , x9 là các nghiệm
phân biệt của P( x).

Giả sử tồn tại x0 sao cho P ( x0 )  0 thì Q ( x0 )  0 , suy ra Q( x0 )  1.

Ta có

P( x0 )  ( x0  x1 )( x0  x2 ) ( x0  x9 ) Q( x0 )  1 (1)  2  (2)  3  (3)  4  (4)  5  2880  2017 .

Điều mâu thuẫn này cho thấy điều giả sử là sai. Ta có đpcm.
Bài 4. (Belarus)
Cho   (0;1] , chứng minh rằng không tồn tại dãy số dương ( xn ) gồm vô hạn số hạng sao cho

xn  2   xn1  xn với mọi n  1, 2,3,

Lời giải.
Giả sử tồn tại dãy số vô hạn thỏa mãn đề.
Ta có  xn 1  xn  0, n  xn   xn 1  xn 1 .
2
x  
Suy ra  xn 1  xn  xn1    n  xn 1  2 xn nên xn    hay dãy ( xn ) bị chặn.
xn 1 2

Do dãy đã cho là vô hạn nên tồn tại lim xn  L  0 (vì dãy tăng).

Khi đó L  L (  1)  L    1  0 , mâu thuẫn. Vậy điều giả sử ban đầu là sai.

Bài 5. (Moldova)

Cho P ( x )  an x n  an 1 x n 1    a2 x 2  a1 x  a0 cho an  0 và a0  max a1 , a2 , , an  . Gọi b là


hệ số của x n 1 , chứng minh rằng P 2 (1)  4b .

Lời giải.
Ta có b  an a1  an 1a2    a1an và

P 2 (1)  ( an  an 1    a1  a0 ) 2  4a0 (an  an 1    a1 )  4b.

Bài 6. (Ấn Độ)

2
Cho đa thức bậc ba P ( x )  x 3   x  4  2  2016n với n  . Biết rằng P có ba nghiệm tự nhiên
(không nhất thiết phân biệt), chứng minh rằng   3.
Lời giải. Giả sử phương trình có ba nghiệm là a, b, c   và n  0 thì

abc  4  2  2016n  4(mod 7) .

Mặt khác a  b  c  0 nên a 3  b3  c 3  3abc  5(mod 7) , mà abc  0(mod 7) nên số dư của


a 3 , b3 , c 3 khi chia cho 7 thuộc tập 1, 6 nên điều trên không thể xảy ra.

Vậy n  0 hay abc  2, a  b  c  0 ; dễ dàng tính được (a, b, c)  (2, 1, 1) hay   3.

Bài 7. (ELMO)

 1
Cho k   0;  và a, b  (0;1). Xét dãy số
 2

 1  an
an1  bn 1  (bn ) k
 2 và  .
a0  a b
 0  b

Chứng minh rằng tồn tại n để an  bn .

a 1 n
Lời giải. Ta tính được an  1  n
và bn  b k . Ta đưa về chứng minh
2

a 1 kn  a 1 
1 n
 b  ln  1  n   k n  ln b .
2  2 

Bằng khảo sát hàm số, ta có


ln(1  x)   x với mọi x  (0;1) .

a 1 a 1
Ta lại đưa về chứng minh n
 k n  ln b  (2k ) n  .
2 ln b
a 1
Chú ý rằng 0  2k  1 và  0 nên sẽ tồn tại n đủ lớn để khẳng định trên là đúng.
ln b
Bài 8. (Benelux)

Cho số nguyên dương k  2 và đặt 65k  an an 1  a2 a1a0 . Xét đa thức

P ( x )  an x n  an 1 x n 1    a1 x  a0 .

Chứng minh rằng đa thức này không có nghiệm hữu tỷ.

3
Lời giải.
Trước hết, ta sẽ chứng minh bổ đề:
p
Nếu đa thức f ( x)  [ x] và có nghiệm là x  với ( p, q)  1 thì f ( x) có thể viết thành
q

f ( x)  (qx  p) g ( x) mà g ( x)  [ x].

Bổ đề này được suy ra trực tiếp từ kết quả sau: Tích của hai đa thức nguyên bản là một đa thức
nguyên bản (đa thức nguyên bản – premitive polynomial là đa thức hệ số nguyên mà các hệ số
không có ước nguyên tố chung).
n m
Thật vậy, xét f ( x)   ai x i và g ( x)   bi x i là hai đa thức nguyên bản; giả sử tích f ( x) g ( x)
i0 i0

là một đa thức không nguyên bản, tức là có một ước nguyên tố chung p cho các hệ số.

Do f , g nguyên bản nên tồn tại ar và bs không chia hết cho p .

Giả sử rằng r là chỉ số lớn nhất trong f và s là chỉ số lớn nhất trong g thỏa mãn điều kiện đó.
Khi đó p | ai , p | b j với i  r , j  s. Xét lũy thừa x r  s với hệ số là 
i j r  s
ai b j .

Ta thấy nếu i  r thì hệ số ai b j sẽ chia hết cho p ; còn nếu i  r thì j  s nên hệ số đó cũng chia
hết cho p ; trong khi đó chỉ có ar bs là không chia hết cho p nên tổng trên không chia hết cho p,
mâu thuẫn.

Áp dụng vào bổ đề, ta thấy nếu g ( x) không có hệ số nguyên thì tồn tại m  1 và m    sao cho
mg ( x )  [ x ] và mg ( x ) là đa thức nguyên bản; mà qx  p cũng nguyên bản nên suy ra mf ( x )
nguyên bản, vô lý. Suy ra g ( x)  [ x].

Quay trở lại bài toán,


Dễ thấy các nghiệm thực của P ( x) đều âm vì tất cả hệ số của P ( x) âm. Giả sử P ( x) có nghiệm
p
là x   với p, q    và ( p, q)  1. Khi đó
q

P( x)  (qx  p)Q( x) với Q( x)  [ x].

Suy ra 65k  (10q  p )Q (10) . Chú ý rằng a1  2, a0  5 vì 65k với k  2 tận cùng là 25.

Ta cũng có p | a0  5 nên p  1 hoặc p  5. Ta xét hai trường hợp:

Nếu p  1 thì 10q  1 là ước của 65k , nhưng q  1, 2, 3, ,9 và 65k không có ước nào có dạng
10q  1 như trên nên vô lý.

4
Nếu p  5 thì 10q  5 là ước của 65k hay 2q  1 là ước của 13k  5k 1 .

Vì 3  2q  1  19 nên 2q  1  5,13 và q  2, 6 . Suy ra an là số chẵn vì q | an .

5  5
(1) Nếu q  6 thì x   là nghiệm của P ( x) nên P     0 hay
6  6

an  5  an 1  5   6    a2  5   6 n  2  2   5   6n 1  5  6n  0 .
n n 1 2

Chia hai vế cho 5 , ta có

an   5   an 1  5   6    a2  5   6n  2  2  6n 1  6n  0 .
n 1 n2

Dựa vào chữ số tận cùng, ta suy ra 2  6n 1  6n  4  6n 1 tận cùng là 0 , vô lý.

5  5
(2) Nếu q  2 thì x   là nghiệm của P ( x) nên P     0 hay
2  2

an  5  an 1  5   2    a2  5  2 n  2  2   5   2 n 1  5  2n  0 nên


n n 1 2

an  5   an 1  5   2    a2  5   2n  2  0 .
n n 1 2

Lại chia hai vế cho (5) 2 , ta có

an   5   an 1  5 
n2 n 3
 2    a3 ( 5)  2 n 3  a2  2 n  2  0 .

Nếu k  3 thì 625 | 65k nên 65k sẽ có tận cùng là 625 và a2  6 nên vế trái có tận cùng khác 0 ,
không thỏa. Suy ra k  2. Khi đó, ta có P ( x )  4 x 3  2 x 2  2 x  5 , dễ dàng kiểm tra trực tiếp được
đa thức này không có nghiệm hữu tỷ.
Vậy với mọi k  2 thì đa thức P ( x) xác định như trên không có nghiệm hữu tỷ.

Bài 9. (Iran MO)


a) Chứng minh rằng không tồn tại dãy vô hạn các số nguyên dương a1 , a2 , a3 , sao cho

gcd( ai  j , a j  i )  1 với mọi i  j.

b) Chứng minh rằng tồn tại dãy số (an ) gồm vô hạn các số nguyên dương sao cho

gcd( ai  j , a j  i ) không chia hết cho 2017.

Lời giải.
a) Giả sử tồn tại dãy thỏa mãn đề bài. Xét số a2n , ta có hai trường hợp:

5
(1) Nếu a2 n  2m  1 là số lẻ. Xét a2 n 1 , ta phải có

gcd( a2 n 1  2n, a2 n  2n  1)  1 hay gcd(a2 n 1  2n, 2m  2 n)  1 , suy ra a2 n 1 lẻ.

Tương tự thì a2 n 1 cũng lẻ. Lại xét a2 n 1 và a2 n 1 (là các số cùng lẻ):

gcd(a2 n 1  2n  1, a2 n 1  2n  1)  2 , mâu thuẫn.

(2) Nếu a2 n  2m là số chẵn. Xét a2 n  2 , ta phải có

gcd(a2 n  2  2n, a2 n  2n  2)  1 nên a2 n  2 lẻ, ta đưa về trường hợp ở trên.

Vậy không tồn tại dãy số thỏa mãn đề bài.


b) Chọn dãy số an  n  1 (mod 2017) thì dễ thấy

ai  j  i  j  1(mod 2017) và a j  i   j  i  1(mod 2017) .

Nếu cả hai số trên chia hết cho 2017 thì 2017 | 2 , vô lý. Do đó, dãy như trên thỏa mãn đề bài.

Bài 10. (China MO)


n
Với số nguyên dương n , xét đa thức P ( x)   ai x i , trong đó ai  0;1 và ai  Cni (mod 2) . Hỏi
i0

có bao nhiêu số n  2017 sao cho P( x) có thể phân tích thành tích của 5 đa thức hệ số nguyên?

Lời giải.
Ta có bổ đề:

(1) (Định lý Lucas) Với m, n    , đặt m   2a , n   2b với A, B là tập hợp các số tự nhiên.
a A bB

Khi đó, C lẻ khi và chỉ khi B  A.


n
m

n
(2) Với mọi số nguyên dương n thì P ( x)  x 2  1 bất khả quy. Ta có thể chứng minh bằng cách
xét P( x  1) và chú ý rằng C2i k là số chẵn với 1  i  2k  1.

Ta viết một số đa thức trong đề bài đã cho


P1 ( x)  x  1
P2 ( x)  x 2  1
P3 ( x)  x3  x 2  x  1  ( x  1)( x 2  1)  P1 ( x)  P2 ( x)
P4 ( x)  x 4  1
P5 ( x)  x5  x 4  x  1  ( x  1)( x 4  1)  P1 ( x)  P4 ( x).

6
Ta sẽ chứng minh rằng nếu n  2a1  2a2    2ak với 0  a1  a2    ak thì

Pn ( x )  Pa1 ( x )  Pa2 ( x ) Pak ( x )  ( x 2  1)( x 2  1)  ( x 2  1) .


a1 a2 ak

Thật vậy,

Theo bổ đề 1 thì Pn ( x ) chính là tổng của các x b mà b là tổng các phần tử trong một tập con tùy
 
ý của 2 a1 , 2a2 , , 2ak .

Mặt khác, vế phải có tất cả k thừa số và khi khai triển ra, có tất cả 2k số hạng. Mỗi số hạng có
 
lũy thừa là tổng của một nhóm tùy ý các phần tử lấy từ 2 a1 , 2a2 , , 2ak , điều này cho thấy đẳng
thức trên là đúng.

Vậy các số n cần tìm sẽ có 5 chữ số trong biểu diễn nhị phân, mà 2017  11111100001(2) nên chỉ
cần chọn 5 vị trí tùy ý trong 11 vị trí trên rồi đặt vào đó các số 1 (các vị trí còn lại đặt số 0 ) thì
ta sẽ có một số thỏa mãn đề bài; vì thế nên số lượng cần tìm là C115 .

Bài 11. (USA TST)


Cho các đa thức f , g có hệ số không âm thỏa mãn

f ( x)
x 2  cx  1  với mọi x ,
g ( x)

trong đó c  2  2  2 . Chứng minh rằng deg f  16.

Lời giải.

Giả sử deg f  n  1. Đặt g ( x )  a0  a1 x  a2 x 2    an  2 x n  2 với ai  0 và an  2  0.

2
Chú ý rằng c  2 cos .
16
Đồng nhất hệ số, ta phải có các bất đẳng thức sau

     
a1  2 cos 16  a0  a1  sin 16  2 cos 16  sin 16 a0
 
a  a  2 cos   a ( a  a )sin 2  2 cos   sin 2 a
 0 2
16
1
 0 2
16 16 16
1

 
  
   (n  2)  ( n  2)
an  4  an  2  2 cos  an 1 ( an  4  an  2 )  sin  2 cos  sin an  3
 16  16 16 16
   (n  1)  (n  1)
an 3  2 cos  an  2  an 3  sin  2 cos  sin an  2
 16  16 16 16

7
k
Nhân bất đẳng thức thứ k cho sin với công thức biến đổi tích thành tổng:
16
 ( k  1) k ( k  2)
2 cos sin  sin  sin , ta được
16 16 16 16

  2
a1  sin  sin a0
 16 16
 2   3 
(a0  a2 ) sin   sin  sin  a1
 16  16 16 

 (n  2)  (n  1) (n  3)
(an  4  an  2 )  sin 
  sin  sin  an 3
 16  16 16 

an 3  sin (n  1)   sin (n  2)  sin n  an 2
 16  16 16 

Cộng tất cả các đánh giá trên lại, ta được

(n  2)  ( n  2) n 
sin  an  2   sin  sin  an  2 hay
16  16 16 
n k
sin  0 , suy ra n  16 (vì sin  0, k  0,1, 2, ,15 ).
16 16
Vậy ta đã chứng minh được deg f  16.

Bài 12. (China TST)


Cho đa thức P( x )  [ x ] có bậc ba và ba nghiệm vô tỷ a, b, c có tổng là 0. Giả sử tồn tại p, q 
sao cho a  b 2  pb  q . Chứng minh rằng T  p 2  2 p  4q  7 là số chính phương.

Lời giải.

Bước 1. Đặt Q ( x )  x 2  px  q thì theo giả thiết, ta có

P (Q (b))  P(a )  0 .

Suy ra P(Q( x)) có nghiệm là x  b . Xét phép chia

P(Q( x))  P( x)  f ( x)  r ( x) với deg r ( x)  2 .

Thay x  b vào, suy ra r (b)  0 . Nếu deg r  0 thì lại xét phép chia

P ( x)  f1 ( x)  r ( x )  r1 ( x ) với deg r1 ( x )  1 .

Chú ý rằng r1 ( x )  [ x] và r1 (b)  0 với b vô tỷ nên vô lý, suy ra deg r  0 hay

8
P(Q( x))  P( x)  f ( x) .

Điều này cho thấy P ( x) là đa thức hệ số nguyên bậc nhỏ nhất nhận x  b là nghiệm. Từ đây ta
cũng có a, b, c phải phân biệt; vì nếu không, giả sử a  b thì P( x) là một đa thức bậc hai hệ số
nguyên có nghiệm là x  a , không thỏa. Từ đây ta có P(Q(a))  P(Q(c))  0.

Suy ra Q ( a )  a, b, c . Ta xét các trường hợp:

Nếu Q(a)  a thì a  a 2  pa  q , dễ dẫn đến điều vô lý tương tự trên (vì).

Nếu Q(a)  b thì a 2  pa  q  b , mà b 2  pb  q  a nên

a  b  (b  a)(a  b)  p (b  a )  1  (a  b)  p

nên p  c  1 , là số vô tỷ, mâu thuẫn.

Do đó, Q(a)  c và tương tự Q(c)  b. Ta có hệ

a  b 2  pb  q

b  c  pc  q (*)
2

c  a 2  pa  q

Bước 2. Trừ từng vế hai phương trình đầu của (*) , ta có

a  b  b 2  c 2  p (b  c)  (b  c)(b  c  p )  (b  c )( p  a ) .

Tương tự thì b  c  (c  a )( p  b) và c  a  (a  b)( p  c) nên nhân tất cả các hệ thức lại thì

( p  a)( p  b)( p  c)  1.

Cộng các đẳng thức trong (*) lại, ta được

3
a 2  b 2  c 2  3q  0  ab  bc  ca  q.
2
Nhân phương trình thứ 1, 2,3 của (*) cho b, c, a rồi cộng lại, ta có

ab  bc  ca  a 3  b3  c 3  p(a 2  b 2  c 2 )
3q q
  3abc  3 pq  abc   pq
2 2
Thay tất cả vào đẳng thức ( p  a)( p  b)( p  c)  1, ta được

9
p3  p 2 (a  b  c)  p (ab  bc  ca)  abc  1
3 pq q
 p3    pq  1
2 2
q  1

 q  2( p  p  1)
2

1 3 1
Nếu q  1 thì abc   p nên có đa thức P ( x )  x 3  x   p , đây là hàm đồng biến nên không
2 2 2
thể có ba nghiệm thực phân biệt được.

Do đó q  2( p 2  p  1) nên

T  p 2  2 p  8( p 2  p  1)  7  9 p 2  6 p  1  (3 p  1) 2 là số chính phương.

Ta có đpcm.
Bài 13. Cho dãy số nguyên dương phân biệt (an ) thỏa mãn:

i) Với mọi n thì an  n.

ii) a1  5, a2  4, a3  3 .

a) Chứng minh rằng tồn tại n  2017 sao cho an  n  1 ?

b) Giả sử an  n  2 với mọi n  2017 , hỏi có tất cả bao nhiêu dãy số như thế?

Lời giải.

a) Bài toán có thể giải quyết dễ dàng bằng phản chứng và Dirichlet. Thật vậy, nếu an  n  1 với
mọi n  2017 thì các số hạng a4  a2017 sẽ nhận các giá trị trong tập hợp 6  2018 . Khi đó, sẽ
có hai số hạng bằng nhau, không thỏa.

b) Nếu đã có an  n  2 với mọi n  2017 thì các số hạng a4  a2017 sẽ nhận các giá trị trong
tập hợp 6  2019. Nhận xét:

a2017  2017, 2018, 2019 nên có 3 cách chọn.

a2016  2016, 2017, 2018, 2019 nhưng vì a2017 đã lấy một số nên cũng còn 3 cách chọn.

Tương tự, đến a6 vẫn có 3 cách chọn. Còn lại a5 có 2 cách chọn và a4 có 1 cách chọn.

Theo nguyên lý nhân, ta có 2  32012 dãy thỏa mãn.

10

You might also like